What is Inequality: Definition and 1000 Discussions

In mathematics, the triangle inequality states that for any triangle, the sum of the lengths of any two sides must be greater than or equal to the length of the remaining side. This statement permits the inclusion of degenerate triangles, but some authors, especially those writing about elementary geometry, will exclude this possibility, thus leaving out the possibility of equality. If x, y, and z are the lengths of the sides of the triangle, with no side being greater than z, then the triangle inequality states that




z

x
+
y
,


{\displaystyle z\leq x+y,}
with equality only in the degenerate case of a triangle with zero area.
In Euclidean geometry and some other geometries, the triangle inequality is a theorem about distances, and it is written using vectors and vector lengths (norms):






x

+

y





x


+


y


,


{\displaystyle \|\mathbf {x} +\mathbf {y} \|\leq \|\mathbf {x} \|+\|\mathbf {y} \|,}
where the length z of the third side has been replaced by the vector sum x + y. When x and y are real numbers, they can be viewed as vectors in R1, and the triangle inequality expresses a relationship between absolute values.
In Euclidean geometry, for right triangles the triangle inequality is a consequence of the Pythagorean theorem, and for general triangles, a consequence of the law of cosines, although it may be proven without these theorems. The inequality can be viewed intuitively in either R2 or R3. The figure at the right shows three examples beginning with clear inequality (top) and approaching equality (bottom). In the Euclidean case, equality occurs only if the triangle has a 180° angle and two 0° angles, making the three vertices collinear, as shown in the bottom example. Thus, in Euclidean geometry, the shortest distance between two points is a straight line.
In spherical geometry, the shortest distance between two points is an arc of a great circle, but the triangle inequality holds provided the restriction is made that the distance between two points on a sphere is the length of a minor spherical line segment (that is, one with central angle in [0, π]) with those endpoints.The triangle inequality is a defining property of norms and measures of distance. This property must be established as a theorem for any function proposed for such purposes for each particular space: for example, spaces such as the real numbers, Euclidean spaces, the Lp spaces (p ≥ 1), and inner product spaces.

View More On Wikipedia.org
  1. B

    MHB Maximum volume using AM GM inequality

    Hi everyone, I'm a bit confused with this question. An airline demands that all carry-on bags must have length + width + height at most 90cm. What is the maximum volume of a carry-on bag? How do you know this is the maximum? [Note: You can assume that the airline technically mean "all carry...
  2. D

    MHB Prove AM-GM Inequality: What Values to Use?

    Could someone please help me with this question: What values are we meant to use to prove this inequality? many thanks
  3. kaliprasad

    MHB Proving an Inequality: $(\sin\, x)^{\sin\, x}\,<(\cos\, x)^{\cos\, x}$

    One of the 2 inequalities $(\sin\, x)^{\sin\, x}\,<(\cos\, x)^{\cos\, x} $ and $(\sin\, x)^{\sin\, x}\,>(\cos\, x)^{\cos\, x} $ is always true for all x such that $0 \,< \, x \, < \pi/4$ Identify the inequality and prove it
  4. anemone

    MHB Can You Prove the Inequality Challenge VI for Arctan Sequences?

    If $\alpha_n=\arctan n$, prove that $\alpha_{n+1}-\alpha_n<\dfrac{1}{n^2+n}$ for $n=1,\,2,\,\cdots$.
  5. anemone

    MHB Inequality Challenge V: Prove $(a+b)^{a+b} \le (2a)^a(2b)^b$

    Prove that for any real numbers $a$ and $b$ in $(0,\,1)$, that $(a+b)^{a+b}\le (2a)^a(2b)^b$.
  6. D

    Inequality with absolute values

    Wonder if this is true or just mistype: |x+y| \leq |x| +|y| If this is true how to proof because cannot find it out anywhere written Regards
  7. Q

    Can Complex Numbers be Compared Using Greater-Than and Less-Than Relations?

    Are the less than (<) and greater than(>) relations applicable among complex numbers? By complex numbers I don't mean their modulus, I mean just the raw complex numbers.
  8. Saitama

    MHB Inequality with area of triangle

    Problem: If A is the area and 2s the sum of three sides of a triangle, then: A)$A\leq \frac{s^2}{3\sqrt{3}}$ B)$A=\frac{s^2}{2}$ C)$A>\frac{s^2}{\sqrt{3}}$ D)None Attempt: From heron's formula: $$A=\sqrt{s(s-a)(s-b)(s-c)}$$ From AM-GM: $$\frac{s+(s-a)+(s-b)+(s-c)}{4}\geq...
  9. J

    Is Tr(ABAB) Nonnegative for Symmetric Matrices A and B?

    Here's the claim: Assume that A and B are both symmetric matrices of the same size. Also assume that at least other one of them does not have negative eigenvalues. Then \textrm{Tr}(ABAB)\geq 0 I don't know how to prove this!
  10. anemone

    MHB Can We Prove This Inequality Challenge IV?

    Prove that $\dfrac{1}{\sqrt{4x}}\le\left( \dfrac{1}{2} \right)\left( \dfrac{3}{4} \right)\cdots\left( \dfrac{2x-1}{2x} \right)<\dfrac{1}{\sqrt{2x}}$.
  11. anemone

    MHB Can Jensen's Inequality Solve the Inequality Challenge III?

    Show that $e^\dfrac{1}{e}_{\phantom{i}}+e^{\dfrac{1}{\pi}}_{\phantom{i}} \ge2e^{\dfrac{1}{3}}_{\phantom{i}}$.
  12. O

    MHB How Can I Prove Gronwall's Inequality?

    i am asked to prove phi(t) ≤ phi(s)e^(c(|t-s|)) for t in I. I have to proof this how can I start supposing that t>s and that d/dt(e^(-ct) phi(t))
  13. M

    MHB Is it True That for any Pythagorean Triple, $(\frac ca + \frac cb)^2 > 8$?

    Let $(a,b,c)$ be a Pythagorean triple, specifically, a triplet of positive integers with property $a^2 + b^2 = c^2$. Show that $(\frac ca + \frac cb)^2 > 8$. EDIT: Added a small clarification.
  14. M

    MHB Proof of Inequality: $x^2 + xy^2 + xyz^2 \geq 4xyz - 4$

    Let $x,y,z$ be positive real numbers. Show that $x^2 + xy^2 + xyz^2 \geq 4xyz - 4$.
  15. anemone

    MHB What is the smallest possible value of y for given x and y?

    Let $x,\,y$ be positive integers such that $\dfrac{7}{10}<\dfrac{x}{y}<\dfrac{11}{15}$. Find the smallest possible value of $y$.
  16. K

    Proving ac < bd: Inequality Proof

    If 0 ≤ a < b and 0 ≤ c <d, then prove that ac < bd I have taken the proof approach from some previous problems in Spivak's book on Calculus (3rd edition). This is problem 5.(viii) in chapter 1: Basic Properties of Numbers. I did as follows: If a = 0 or c = 0, then ac = 0, but since...
  17. M

    MHB Prove Geometry Inequality: 60° ≤ ($aA$+$bB$+$cC$)/($a$+$b$+$c$) < 90°

    (BMO, 2013) The angles $A$, $B$, $C$ of a triangle are measures in degrees, and the lengths of the opposite sides are $a$,$b$,$c$ respectively. Prove: \[ 60^\circ \leq \frac{aA + bB + cC}{a + b + c} < 90^\circ. \] Edit: Update to include the degree symbol for clarification. Thanks, anemone.
  18. anemone

    MHB Is it possible to prove the inequality without using induction?

    Show that $\dfrac{1}{2} \cdot \dfrac{3}{4} \cdot \dfrac{5}{6} \cdots \dfrac{1997}{1998} >\dfrac{1}{1999}$, where the use of induction method is not allowed.
  19. P

    Help with use of Chebyshev's inequality and sample size

    Homework Statement Homework Equations P (|Y - μ| < kσ) ≥ 1 - Var(Y)/(k2σ2) = 1 - 1/k2 ?? The Attempt at a Solution using the equation above 1 - 1/k2 = .9 .1 = 1/k2 k2 = 10 k = √10 = 3.162 k = number of standard deviations. After this I don't know where to go...
  20. B

    Integration inequality proof validation

    Homework Statement Let ##f:[a,b]\rightarrow\mathbb{R}## and ##g:[a,b]\rightarrow\mathbb{R}## be continuous functions having the property ##f(x)\leq g(x)## for all ##x\in[a,b]##. Prove ##\int_a^b \mathrm f <\int_a^b\mathrm g## iff there exists a point ##x_0## in ##[a,b]## at which...
  21. Albert1

    MHB What values of tan alpha and tan beta satisfy a trigonometric inequality?

    $0<\alpha < \dfrac {\pi}{2}$ $0<\beta < \dfrac {\pi}{2}$ prove: $(1): \,\, \dfrac{1}{ \cos^2 \alpha}+ \dfrac {1}{ \sin^2 \alpha \, \sin^2 \beta \, \cos^2 \beta} \geq 9 $ determine the values of $ \tan \alpha$ and $ \tan \beta $ when : $(2): \: \dfrac{1}{ \cos^2 \alpha}+ \dfrac {1}{ \sin^2 \alpha...
  22. Albert1

    MHB Prove Inequality: $ab+c, bc+a, ca+b \geq 18$

    $a,b,c\geq 1$ prove :$\dfrac {ab+c}{c+1}+\dfrac {bc+a}{a+1}+\dfrac {ca+b}{b+1}\geq\dfrac {18} {a+b+c+3}$
  23. J

    Proof of a probability inequality

    Homework Statement I'm working on some MIT OCW (http://ocw.mit.edu/courses/electrical-engineering-and-computer-science/6-436j-fundamentals-of-probability-fall-2008/assignments/MIT6_436JF08_hw01.pdf). I've attempted problem #5, just looking for some comments on the quality / validity of my...
  24. T

    Solving an Inequality: |x-3| < 2|x|

    Homework Statement Solve the given inequality by interpreting it as a statement about distances in the real line: |x-3| < 2|x| Homework Equations The Attempt at a Solution I have no clue what to do here and I do not understand the answer in the textbook Goes something like...
  25. P

    Why correct this inequality log(d/d-1)>(1/d)?

    Why correct this inequality, log(d/(d-1))>1/d for d≥2?
  26. B

    Proving an Inequality: Understanding α - 2β < 0 with β > α

    Hello, I am given that β > α, which can be written as β - α > 0. What justification would I have to use in order to conclude that α - 2β < 0, given that the preceding propositions are true? Could someone possibly help me?
  27. B

    MHB How is this inequality obtained?

    Introduction to Operator Theory and Invariant Subspaces - B. Beauzamy - Google BooksIn page 144 of this preview I don't know how they obtain the inequality in (1). It looked like cauchy schwarz but I don't think it is. I also don't know how they connect the norm of the integral to the supremum...
  28. anemone

    MHB Can you prove this inequality challenge involving positive integers?

    Let $a$ and $b$ be positive integers. Show that $\dfrac{(a+b)!}{(a+b)^{a+b}}\le \dfrac{a! \cdot b!}{a^ab^b}$.
  29. A

    Is it possible to prove this set inequality given the constraints?

    Homework Statement Homework Equations I have to use these set identities: The Attempt at a Solution Pretty sure this is impossible since it's an inequality.
  30. anemone

    MHB Can You Prove This Fraction Sequence is Less Than 1/1000?

    Show that $\dfrac{1}{2}\cdot\dfrac{3}{4}\cdot\dfrac{5}{6} \cdots\dfrac{999999}{1000000}<\dfrac{1}{1000}$
  31. S

    Proving the Inequality: x^4+x^3y+x^2y^2+xy^3+y^4 > 0 for x,y>0

    Prove that if x and y are not both , then x^4+x^3y+x^2y^2+xy^3+y^4 > 0 I have no idea how to start this proof, can anyone give me an idea?
  32. A

    Solve the inequality for x, given that (4x - 16) / [(x - 3)(x - 9)] <

    Homework Statement Solve the inequality for x, given that (4x - 16) / [(x - 3)(x - 9)] < 0 Homework Equations I can't think of any for this type of problem... The Attempt at a Solution (4x - 16) / [(x - 3)(x - 9)] < 0 4(x - 4) / [(x - 3)(x - 9)] < 0 I'm not sure where...
  33. anemone

    MHB How can positive numbers be used to prove an inequality challenge?

    If $a,\,b,\,c$ are positive numbers, show that $8(a^3+b^3+c^3)\ge (a+b)^3+(a+c)^3+(b+c)^3$.
  34. R

    (Complex analysis). Show that the inequality holds

    Homework Statement Show that the inequality\left|\frac{z^2-2z+4}{3x+10}\right|\leq3holds for all z\in\mathbb{C} such that |z|=2 Homework Equations Triangle inequality The Attempt at a Solution I'm not really sure how to go about this. the x is throwing me off. Should I write it out with...
  35. Quarlep

    Mathematical Solution in Inequality

    Hi I have a problem about Inequality. Let's suppose we have a inequality like this: axb≥d and cxd≥d so I want to find connection between a and d its possible to do something like that Thanks
  36. A

    What is the value of M when |x|≤ 3 in the inequality (x^2+2x+1)/(x^2+3) ≤ M?

    1. |(x2+2x+1)/(x2+3)|≤ M. Find the value of M when |x|≤ 3. 2. |u+v|=|u|+|v| 3. I understand that you start off by distributing the absolute value symbols into the individual terms as above. Then you maximize the numerator, using 3 as the value for x. However, my professor then...
  37. M

    MHB Cauchy-Schwarz inequality for pre-inner product

    Dear all, I've encountered some problems while looking through the book called "Operator Algebras" by Bruce Blackadar. At the very beginning there is a definition of pre-inner product on the complex vector space: briefly, it's the same as the inner product, but the necessity of x=0 when [x,x]=0...
  38. tomwilliam2

    Proof by contradiction for simple inequality

    Homework Statement I'm trying to show that if ##a \approx 1##, then $$-1 \leq \frac{1-a}{a} \leq 1$$ I've started off trying a contradiction, i.e. suppose $$ \frac{|1-a|}{a} > 1$$ either i) $$\frac{1-a}{a} < -1$$ then multiply by a and add a to show $$1 < 0$$ which is clearly...
  39. K

    MHB Trigonometric inequality bounded by lines

    How can it be shown that $$16x\cos(8x)+4x\sin(8x)-2\sin(8x)<|17x|?$$ This problem arises from work with damped motion in spring-mass systems in Differential Equations. I have gotten to this inequality after some algebraic manipulation, but am completely stuck here. Here is the illustrative...
  40. C

    Triangle inequality proof in Spivak's calculus

    So hi, there's one little thing which I'm not understanding in the proof. After the inequality Spivak considers the two expressions to be equal. Why?!? I just don't see why we can't continue with the inequality and when we have factorized the identity to (|a|+|b|)^2 we can just replace...
  41. anemone

    MHB How can AM-GM be used to solve the Inequality Challenge II?

    Prove that $\sqrt[3]{\dfrac{2}{1}}+\sqrt[3]{\dfrac{3}{2}}+\cdots+\sqrt[3]{\dfrac{996}{995}}-\dfrac{1989}{2}<\dfrac{1}{3}+\dfrac{1}{6}+\cdots+ \dfrac{1}{8961}$
  42. anemone

    MHB Prove Inequality: $\dfrac{1}{a}+\dfrac{1}{b}+\dfrac{1}{c}+\dfrac{1}{d}$

    Prove that $\dfrac{1}{\dfrac{1}{a}+\dfrac{1}{b}}+\dfrac{1}{ \dfrac{1}{c}+\dfrac{1}{d}} \le \dfrac{1}{\dfrac{1}{a+c}+\dfrac{1}{b+d}}$ for all positive real numbers $a, b, c, d$.
  43. C

    Understanding the Norm Inequality ||Av|| ≤ ||A||||v||

    Hi, With the following norm inequality: ||Av|| ≤ ||A||||v|| implies ||A|| = supv [ ||Av||/||v|| ] I understand that sup is the upper bound of a set B, or least upper bound if B is a subset of A, where the upper bounds are elements of both B and A. Is this saying that the norm of A...
  44. T

    Solve Inequality Problem: Sum of a <= 100

    Let a, b, and c be positive integers such that: \frac{1}{a^2} + \frac{1}{b^2} = \frac{1}{c^2} Find the sum of all possible values of a that are less than or equal to 100. My shot at it: I think may be that equation would also equals to a^2 + b^2 = c^2... So may be a is the sum of all the...
  45. Albert1

    MHB Triangle Inequality Proof for Side Lengths of Triangle ABC

    Triangle ABC with side lengths a,b,c please prove : $ \sqrt {ab}+\sqrt {bc}+\sqrt {ca}\leq a+b+c<2\sqrt {ab}+2\sqrt {bc}+2\sqrt {ca}$
  46. R

    Uncovering the Flaws of Born Inequality: A Deeper Look into Quantum Particles

    The Born in ineqality fails experimentally and fails to make sense if we think of quanta as particles. Does it make sense otherwise?
  47. A

    Proving the Inequality of e^x Using Taylor's Theorem

    Homework Statement Show that if 0 \le x \le a, and n is a natural number, then 1+\frac{x}{1!}+\frac{x^2}{2!}+...+\frac{x^n}{n!} \le e^x \le 1+\frac{x}{1!}+\frac{x^2}{2!}+...+\frac{x^n}{n!}+\frac{e^ax^{n+1}}{(n+1)!} Homework Equations I used Taylor's theorem to prove e^x is equal to the LHS...
  48. anemone

    MHB Trigonometric Inequality Challenge

    For any triangle $ABC$, prove that $\cos \dfrac{A}{2} \cot \dfrac{A}{2}+\cos \dfrac{B}{2} \cot \dfrac{B}{2}+\cos \dfrac{C}{2} \cot \dfrac{C}{2} \ge \dfrac{\sqrt{3}}{2} \left( \cot \dfrac{A}{2}+\cot \dfrac{B}{2}+\cot \dfrac{C}{2} \right)$
  49. Saitama

    MHB Can the RMS-AM inequality prove the combinatorial coefficient inequality?

    Problem: Prove: $$\sqrt{C_1}+\sqrt{C_2}+\sqrt{C_3}+...+\sqrt{C_n} \leq 2^{n-1}+\frac{n-1}{2}$$ where $C_0,C_1,C_2,...,C_n$ are combinatorial coefficients in the expansion of $(1+x)^n$, $n \in \mathbb{N}$. Attempt: I thought of using the RMS-AM inequality and got...
  50. anemone

    MHB Inequality Challenge: Show $7x+12xy+5y \le 9$

    Let $x, y$ be real numbers such that $9x^2+8xy+7y^2 \le 6$. Show that $7x+12xy+5y \le 9$.
Back
Top